LSAT and Law School Admissions Forum

Get expert LSAT preparation and law school admissions advice from PowerScore Test Preparation.

 Administrator
PowerScore Staff
  • PowerScore Staff
  • Posts: 8916
  • Joined: Feb 02, 2011
|
#81119
Complete Question Explanation

The correct answer choice is (C).

Answer choice (A):

Answer choice (B):

Answer choice (C): This is the correct answer choice.

Answer choice (D):

Answer choice (E):


This explanation is still in progress. Please post any questions below!
User avatar
 Tami Taylor
  • Posts: 38
  • Joined: Jan 03, 2021
|
#83910
Hi PowerScore,

For my first two attempts at this question, I chose (B) and (D). Is (C) the best answer because there is explicit proof in the passage whereas (B) and (D) are sort of "stretch" answers? For example, the passage doesn't speak about who the English thought would better guard their interests and who the English thought possessed the adequate amount of knowledge to rule responsibly. I think the proof for (C) is in this line: "[...] Parliament was unlimited in its power: it could change even the Constitution by its ordinary acts of legislation." Is this correct?

Thank you!
 Adam Tyson
PowerScore Staff
  • PowerScore Staff
  • Posts: 5153
  • Joined: Apr 14, 2011
|
#83926
Correct, Tami! The key is in the question stem: "The author mentions" means you have to find the answer explicitly stated in the text, rather than implied by the author or assumed by the reader. Neither B nor D is ever explicitly stated, so we must reject them.
User avatar
 Tami Taylor
  • Posts: 38
  • Joined: Jan 03, 2021
|
#83934
Awesome. Thanks, Adam!

Get the most out of your LSAT Prep Plus subscription.

Analyze and track your performance with our Testing and Analytics Package.